Can White Castle?Chess position such that with perfect play, mirrors itselfGet 6 pawns in file A or HHow did four chessmen disappear?Reconstruct a game of Loser ChessOne-sided chess: Can you stalemate yourself?Fantastic Foxhole FailuresFind Those Chess Notations! #2That Ain’t Rght… 1# (Definitely!)That Ain’t Right… #3Where is the White King?

Designing a magic-compatible polearm

How long would it take to cross the Channel in 1890's?

Is it illegal to withhold someone's passport and green card in California?

Primes and SemiPrimes in Binary

Shooting someone's past self using special relativity

Has there been any indication at all that further negotiation between the UK and EU is possible?

Encounter design and XP thresholds

I found a password with hashcat, but it doesn't work

Counterfeit checks were created for my account. How does this type of fraud work?

How do I farm creepers for XP without them exploding?

LWC - Local Dev - How can I run the local server on HTTPS?

CircuiTikZ: Start ground relative to the closest component

Can Ogre clerics use Purify Food and Drink on humanoid characters?

How many people are necessary to maintain modern civilisation?

Where's this swanky house and vineyard near a mountain?

Dates on degrees don’t make sense – will people care?

How large would a mega structure have to be to host 1 billion people indefinitely?

I don't like coffee, neither beer. How to politely work my way around that in a business situation?

UK - Working without a contract. I resign and guy wants to sue me

What determines the direction in which motor proteins go?

Constitutionality of U.S. Democratic Presidential Candidate's Supreme Court Suggestion

How to remove this component from PCB

Story about hunting giant lizards for hides on privately owned planet

Ruining the family name



Can White Castle?


Chess position such that with perfect play, mirrors itselfGet 6 pawns in file A or HHow did four chessmen disappear?Reconstruct a game of Loser ChessOne-sided chess: Can you stalemate yourself?Fantastic Foxhole FailuresFind Those Chess Notations! #2That Ain’t Rght… 1# (Definitely!)That Ain’t Right… #3Where is the White King?






.everyoneloves__top-leaderboard:empty,.everyoneloves__mid-leaderboard:empty,.everyoneloves__bot-mid-leaderboard:empty margin-bottom:0;








9












$begingroup$


This is the first chess puzzle I composed in the retrograde genre. I originally posted this in a chess dedicated forum. Hope you like it!



In the following position, is it possible that White could still castle?



enter image description here



  • To prove it's possible, all you have to do is provide a legal game.

  • If you believe it's impossible, you need to provide your reasoning.









share|improve this question









$endgroup$


















    9












    $begingroup$


    This is the first chess puzzle I composed in the retrograde genre. I originally posted this in a chess dedicated forum. Hope you like it!



    In the following position, is it possible that White could still castle?



    enter image description here



    • To prove it's possible, all you have to do is provide a legal game.

    • If you believe it's impossible, you need to provide your reasoning.









    share|improve this question









    $endgroup$














      9












      9








      9


      2



      $begingroup$


      This is the first chess puzzle I composed in the retrograde genre. I originally posted this in a chess dedicated forum. Hope you like it!



      In the following position, is it possible that White could still castle?



      enter image description here



      • To prove it's possible, all you have to do is provide a legal game.

      • If you believe it's impossible, you need to provide your reasoning.









      share|improve this question









      $endgroup$




      This is the first chess puzzle I composed in the retrograde genre. I originally posted this in a chess dedicated forum. Hope you like it!



      In the following position, is it possible that White could still castle?



      enter image description here



      • To prove it's possible, all you have to do is provide a legal game.

      • If you believe it's impossible, you need to provide your reasoning.






      chess retrograde-analysis






      share|improve this question













      share|improve this question











      share|improve this question




      share|improve this question










      asked 8 hours ago









      shoopishoopi

      36519




      36519




















          2 Answers
          2






          active

          oldest

          votes


















          5












          $begingroup$

          Now that we have three increasingly complex proofs (two deleted, one of them mine) that it's impossible, it's pretty clear that it must be




          possible after all!




          Here's why:




          1. b3 Nf6

          2. Bb2 Ng4

          3. Bf6 gxf6

          4. Na3 Ne3

          5. Nc4 Nxf1!

          6. Ne5 fxe5

          7. h4 Rg8

          8. h5 Rg6

          9. hxg6 Bh6

          10. g7 Be3

          11. g8=N Bc5

          12. Nh6 Ba3

          13. Nf5 Ng3

          14. Nd4 exd4

          15. Qb1 Bc1!

          16. Qb2 Nh5

          17. Qc3 dxc3

          18. Rb1 Nf6

          19. Rb2 cxb2

          20. Nf3 b1=R

          21. Nd4 Ra1

          22. Nb5 Ng8

          23. Na3 Bb2+

          24. Nb1 Bg7

          25. e3!! Bf8

          26. 0-0


          In case you haven't already done so, you should totally check out @greenturtle3141's thorough answer (that unfortunately tripped up mere inches before the finish line) to see why the highlighted moves are absolutely essential.




          This is, without doubt, the most refreshing chess problem I've ever tried to solve. Thanks, OP!






          share|improve this answer











          $endgroup$












          • $begingroup$
            Whoa, that's incredible. Nice find, and an amazing puzzle.
            $endgroup$
            – Deusovi
            2 hours ago










          • $begingroup$
            Oh crap, I can't believe that idea actually worked. I've been oofed hard.
            $endgroup$
            – greenturtle3141
            2 hours ago


















          3












          $begingroup$

          (Edit: so this is wrong..)



          Brilliant puzzle! The answer is:




          No! White can't castle.




          [Spoiler alert! Scroll down at your own risk]











          We proceed by contradiction. Assume that indeed, White can castle. We have the following undeniable facts:



          • Neither the White King nor the White h-Rook have moved.

          • Since neither of them have moved, the only way the Black Rook could have gotten to a1 is via a promotion.

          • The only Black pawn that ever moved was the Black g-pawn. This is the piece that promoted.

          We now ask: On which square did this pawn promote? It's not so obvious!



          • The pawn could not have promoted on a1, because this would require 6 captures. Exactly 6 white pieces are missing. That means the pawn captured all of them. But this is impossible because the pawn can only capture on dark squares, and one of the captured pieces would have been a light-squared Bishop.

          • The pawn theoretically could have promoted on b1 via 5 captures.

          • The pawn could not have promoted on c1, for the same reason as that for a1.

          • The path could not have promoted on d1 or f1, because it has to promote to a Rook, and this would check the king, and either the rook would be taken or the King would have to move.

          • Obviously, it could not have promoted on e1.

          • It could not have promoted on g1, because then it cannot get to a1 without the King moving.

          • Obviously, it could not have promoted on h1.

          We conclude that the pawn promoted on b1 via 5 captures. Each of these captures occurred on a dark square, so the five pieces captured were:
          1. The a-Rook
          2. The dark-squared Bishop
          3. The Queen
          4. A Knight
          5. The h-pawn



          The first four captures are easy enough. The problem is the h-pawn. If this pawn stays on its column, it could never be captured by the Black g-pawn in the south-west direction. So it changed columns. To change columns, it must capture a piece. That means that the White h-pawn captured the Black Rook. Even so, this isn't enough to get captured by the Black g-pawn. We conclude that the White h-pawn captured the Black Rook and then promoted on g8 to a capturable piece, say, a second Queen.



          Ok, so the Black g-pawn captures all those 5 pieces. Here's where we're basically at:



          lol



          Notice that I'm keeping the White light-squared Bishop alive. This is very important.



          See, here's the problem now: White has only two movable pieces left: The Knight and the light-squared Bishop. Somehow, Black was able to 1) Promote on b1 to a Rook, 2) Move it to a1, and then 3) White was able to move the White Knight to b1. If the light-squared Bishop was dead, this would be impossible, because the White King would be in check while we were doing all the maneuvering! It couldn't have just been the White Knight blocking the Rook, because then it would be pinned!



          The solution? The White Bishop covers the King by moving to d1. This fact is undeniable, and we will use it very soon.



          (Edit: Could it have instead been a Black piece protecting the White King, say, the Bishop on f8? Actually yes, but this issue resolved itself, see next edit)



          Now we're all set for the main argument. The main problem that is not immediately obvious is that of the last move.



          Looking back at the original board, we first ask: Who moved last? If White moved last, what piece was moved?



          • Clearly it wasn't the b-pawn, because the Black pawn had to get in somehow.

          • It wasn't the e-pawn for sure, because the light-squared Bishop had to get out at some point in order to protect the King from the promoted Rook's check. (Edit:
            So back to the question of if it was a Black piece blocking the check. Well, then, if it was the e-pawn that moved last, then the White light-squared Bishop would be stuck. In short, there wouldn't be enough time for Black to capture this Bishop and run back to its starting square, because White can't move anything else at this point without ruining ability to castle or ruining the Black Rook-White Knight duo on a1 and b1.)

          • It couldn't have been the Knight, because then it would be moving to block the check, i.e. when the Rook got to a1 it checked the king. That means on the move before it was NOT checking the King, so it must have promoted on a1, which we know isn't true.

          • King and Rook are out of the question.

          Thus, Black made the last move. Which piece moved last? Clearly it wasn't that Black rook. And if it was some other piece, it still begs the question: What was White's move before that? This is problematic because if Black's last move was, say, x.. Nb8, then we can apply the same above argument to conclude that White couldn't have made the last move.



          This can only mean that Black captured a White piece on the last move. At this point, it's pretty clear that this can only be the White light-squared Bishop. It's evident that the only square it could have been captured on is g8. But neither the Black f nor h pawns have moved, so this Bishop couldn't possibly have gotten there. Contradiction. Thus, White cannot castle.






          share|improve this answer











          $endgroup$












          • $begingroup$
            See @bass's answer.
            $endgroup$
            – Duck
            1 hour ago











          Your Answer








          StackExchange.ready(function()
          var channelOptions =
          tags: "".split(" "),
          id: "559"
          ;
          initTagRenderer("".split(" "), "".split(" "), channelOptions);

          StackExchange.using("externalEditor", function()
          // Have to fire editor after snippets, if snippets enabled
          if (StackExchange.settings.snippets.snippetsEnabled)
          StackExchange.using("snippets", function()
          createEditor();
          );

          else
          createEditor();

          );

          function createEditor()
          StackExchange.prepareEditor(
          heartbeatType: 'answer',
          autoActivateHeartbeat: false,
          convertImagesToLinks: false,
          noModals: true,
          showLowRepImageUploadWarning: true,
          reputationToPostImages: null,
          bindNavPrevention: true,
          postfix: "",
          imageUploader:
          brandingHtml: "Powered by u003ca class="icon-imgur-white" href="https://imgur.com/"u003eu003c/au003e",
          contentPolicyHtml: "User contributions licensed under u003ca href="https://creativecommons.org/licenses/by-sa/3.0/"u003ecc by-sa 3.0 with attribution requiredu003c/au003e u003ca href="https://stackoverflow.com/legal/content-policy"u003e(content policy)u003c/au003e",
          allowUrls: true
          ,
          noCode: true, onDemand: true,
          discardSelector: ".discard-answer"
          ,immediatelyShowMarkdownHelp:true
          );



          );













          draft saved

          draft discarded


















          StackExchange.ready(
          function ()
          StackExchange.openid.initPostLogin('.new-post-login', 'https%3a%2f%2fpuzzling.stackexchange.com%2fquestions%2f85221%2fcan-white-castle%23new-answer', 'question_page');

          );

          Post as a guest















          Required, but never shown

























          2 Answers
          2






          active

          oldest

          votes








          2 Answers
          2






          active

          oldest

          votes









          active

          oldest

          votes






          active

          oldest

          votes









          5












          $begingroup$

          Now that we have three increasingly complex proofs (two deleted, one of them mine) that it's impossible, it's pretty clear that it must be




          possible after all!




          Here's why:




          1. b3 Nf6

          2. Bb2 Ng4

          3. Bf6 gxf6

          4. Na3 Ne3

          5. Nc4 Nxf1!

          6. Ne5 fxe5

          7. h4 Rg8

          8. h5 Rg6

          9. hxg6 Bh6

          10. g7 Be3

          11. g8=N Bc5

          12. Nh6 Ba3

          13. Nf5 Ng3

          14. Nd4 exd4

          15. Qb1 Bc1!

          16. Qb2 Nh5

          17. Qc3 dxc3

          18. Rb1 Nf6

          19. Rb2 cxb2

          20. Nf3 b1=R

          21. Nd4 Ra1

          22. Nb5 Ng8

          23. Na3 Bb2+

          24. Nb1 Bg7

          25. e3!! Bf8

          26. 0-0


          In case you haven't already done so, you should totally check out @greenturtle3141's thorough answer (that unfortunately tripped up mere inches before the finish line) to see why the highlighted moves are absolutely essential.




          This is, without doubt, the most refreshing chess problem I've ever tried to solve. Thanks, OP!






          share|improve this answer











          $endgroup$












          • $begingroup$
            Whoa, that's incredible. Nice find, and an amazing puzzle.
            $endgroup$
            – Deusovi
            2 hours ago










          • $begingroup$
            Oh crap, I can't believe that idea actually worked. I've been oofed hard.
            $endgroup$
            – greenturtle3141
            2 hours ago















          5












          $begingroup$

          Now that we have three increasingly complex proofs (two deleted, one of them mine) that it's impossible, it's pretty clear that it must be




          possible after all!




          Here's why:




          1. b3 Nf6

          2. Bb2 Ng4

          3. Bf6 gxf6

          4. Na3 Ne3

          5. Nc4 Nxf1!

          6. Ne5 fxe5

          7. h4 Rg8

          8. h5 Rg6

          9. hxg6 Bh6

          10. g7 Be3

          11. g8=N Bc5

          12. Nh6 Ba3

          13. Nf5 Ng3

          14. Nd4 exd4

          15. Qb1 Bc1!

          16. Qb2 Nh5

          17. Qc3 dxc3

          18. Rb1 Nf6

          19. Rb2 cxb2

          20. Nf3 b1=R

          21. Nd4 Ra1

          22. Nb5 Ng8

          23. Na3 Bb2+

          24. Nb1 Bg7

          25. e3!! Bf8

          26. 0-0


          In case you haven't already done so, you should totally check out @greenturtle3141's thorough answer (that unfortunately tripped up mere inches before the finish line) to see why the highlighted moves are absolutely essential.




          This is, without doubt, the most refreshing chess problem I've ever tried to solve. Thanks, OP!






          share|improve this answer











          $endgroup$












          • $begingroup$
            Whoa, that's incredible. Nice find, and an amazing puzzle.
            $endgroup$
            – Deusovi
            2 hours ago










          • $begingroup$
            Oh crap, I can't believe that idea actually worked. I've been oofed hard.
            $endgroup$
            – greenturtle3141
            2 hours ago













          5












          5








          5





          $begingroup$

          Now that we have three increasingly complex proofs (two deleted, one of them mine) that it's impossible, it's pretty clear that it must be




          possible after all!




          Here's why:




          1. b3 Nf6

          2. Bb2 Ng4

          3. Bf6 gxf6

          4. Na3 Ne3

          5. Nc4 Nxf1!

          6. Ne5 fxe5

          7. h4 Rg8

          8. h5 Rg6

          9. hxg6 Bh6

          10. g7 Be3

          11. g8=N Bc5

          12. Nh6 Ba3

          13. Nf5 Ng3

          14. Nd4 exd4

          15. Qb1 Bc1!

          16. Qb2 Nh5

          17. Qc3 dxc3

          18. Rb1 Nf6

          19. Rb2 cxb2

          20. Nf3 b1=R

          21. Nd4 Ra1

          22. Nb5 Ng8

          23. Na3 Bb2+

          24. Nb1 Bg7

          25. e3!! Bf8

          26. 0-0


          In case you haven't already done so, you should totally check out @greenturtle3141's thorough answer (that unfortunately tripped up mere inches before the finish line) to see why the highlighted moves are absolutely essential.




          This is, without doubt, the most refreshing chess problem I've ever tried to solve. Thanks, OP!






          share|improve this answer











          $endgroup$



          Now that we have three increasingly complex proofs (two deleted, one of them mine) that it's impossible, it's pretty clear that it must be




          possible after all!




          Here's why:




          1. b3 Nf6

          2. Bb2 Ng4

          3. Bf6 gxf6

          4. Na3 Ne3

          5. Nc4 Nxf1!

          6. Ne5 fxe5

          7. h4 Rg8

          8. h5 Rg6

          9. hxg6 Bh6

          10. g7 Be3

          11. g8=N Bc5

          12. Nh6 Ba3

          13. Nf5 Ng3

          14. Nd4 exd4

          15. Qb1 Bc1!

          16. Qb2 Nh5

          17. Qc3 dxc3

          18. Rb1 Nf6

          19. Rb2 cxb2

          20. Nf3 b1=R

          21. Nd4 Ra1

          22. Nb5 Ng8

          23. Na3 Bb2+

          24. Nb1 Bg7

          25. e3!! Bf8

          26. 0-0


          In case you haven't already done so, you should totally check out @greenturtle3141's thorough answer (that unfortunately tripped up mere inches before the finish line) to see why the highlighted moves are absolutely essential.




          This is, without doubt, the most refreshing chess problem I've ever tried to solve. Thanks, OP!







          share|improve this answer














          share|improve this answer



          share|improve this answer








          edited 31 mins ago

























          answered 2 hours ago









          BassBass

          32.9k477197




          32.9k477197











          • $begingroup$
            Whoa, that's incredible. Nice find, and an amazing puzzle.
            $endgroup$
            – Deusovi
            2 hours ago










          • $begingroup$
            Oh crap, I can't believe that idea actually worked. I've been oofed hard.
            $endgroup$
            – greenturtle3141
            2 hours ago
















          • $begingroup$
            Whoa, that's incredible. Nice find, and an amazing puzzle.
            $endgroup$
            – Deusovi
            2 hours ago










          • $begingroup$
            Oh crap, I can't believe that idea actually worked. I've been oofed hard.
            $endgroup$
            – greenturtle3141
            2 hours ago















          $begingroup$
          Whoa, that's incredible. Nice find, and an amazing puzzle.
          $endgroup$
          – Deusovi
          2 hours ago




          $begingroup$
          Whoa, that's incredible. Nice find, and an amazing puzzle.
          $endgroup$
          – Deusovi
          2 hours ago












          $begingroup$
          Oh crap, I can't believe that idea actually worked. I've been oofed hard.
          $endgroup$
          – greenturtle3141
          2 hours ago




          $begingroup$
          Oh crap, I can't believe that idea actually worked. I've been oofed hard.
          $endgroup$
          – greenturtle3141
          2 hours ago













          3












          $begingroup$

          (Edit: so this is wrong..)



          Brilliant puzzle! The answer is:




          No! White can't castle.




          [Spoiler alert! Scroll down at your own risk]











          We proceed by contradiction. Assume that indeed, White can castle. We have the following undeniable facts:



          • Neither the White King nor the White h-Rook have moved.

          • Since neither of them have moved, the only way the Black Rook could have gotten to a1 is via a promotion.

          • The only Black pawn that ever moved was the Black g-pawn. This is the piece that promoted.

          We now ask: On which square did this pawn promote? It's not so obvious!



          • The pawn could not have promoted on a1, because this would require 6 captures. Exactly 6 white pieces are missing. That means the pawn captured all of them. But this is impossible because the pawn can only capture on dark squares, and one of the captured pieces would have been a light-squared Bishop.

          • The pawn theoretically could have promoted on b1 via 5 captures.

          • The pawn could not have promoted on c1, for the same reason as that for a1.

          • The path could not have promoted on d1 or f1, because it has to promote to a Rook, and this would check the king, and either the rook would be taken or the King would have to move.

          • Obviously, it could not have promoted on e1.

          • It could not have promoted on g1, because then it cannot get to a1 without the King moving.

          • Obviously, it could not have promoted on h1.

          We conclude that the pawn promoted on b1 via 5 captures. Each of these captures occurred on a dark square, so the five pieces captured were:
          1. The a-Rook
          2. The dark-squared Bishop
          3. The Queen
          4. A Knight
          5. The h-pawn



          The first four captures are easy enough. The problem is the h-pawn. If this pawn stays on its column, it could never be captured by the Black g-pawn in the south-west direction. So it changed columns. To change columns, it must capture a piece. That means that the White h-pawn captured the Black Rook. Even so, this isn't enough to get captured by the Black g-pawn. We conclude that the White h-pawn captured the Black Rook and then promoted on g8 to a capturable piece, say, a second Queen.



          Ok, so the Black g-pawn captures all those 5 pieces. Here's where we're basically at:



          lol



          Notice that I'm keeping the White light-squared Bishop alive. This is very important.



          See, here's the problem now: White has only two movable pieces left: The Knight and the light-squared Bishop. Somehow, Black was able to 1) Promote on b1 to a Rook, 2) Move it to a1, and then 3) White was able to move the White Knight to b1. If the light-squared Bishop was dead, this would be impossible, because the White King would be in check while we were doing all the maneuvering! It couldn't have just been the White Knight blocking the Rook, because then it would be pinned!



          The solution? The White Bishop covers the King by moving to d1. This fact is undeniable, and we will use it very soon.



          (Edit: Could it have instead been a Black piece protecting the White King, say, the Bishop on f8? Actually yes, but this issue resolved itself, see next edit)



          Now we're all set for the main argument. The main problem that is not immediately obvious is that of the last move.



          Looking back at the original board, we first ask: Who moved last? If White moved last, what piece was moved?



          • Clearly it wasn't the b-pawn, because the Black pawn had to get in somehow.

          • It wasn't the e-pawn for sure, because the light-squared Bishop had to get out at some point in order to protect the King from the promoted Rook's check. (Edit:
            So back to the question of if it was a Black piece blocking the check. Well, then, if it was the e-pawn that moved last, then the White light-squared Bishop would be stuck. In short, there wouldn't be enough time for Black to capture this Bishop and run back to its starting square, because White can't move anything else at this point without ruining ability to castle or ruining the Black Rook-White Knight duo on a1 and b1.)

          • It couldn't have been the Knight, because then it would be moving to block the check, i.e. when the Rook got to a1 it checked the king. That means on the move before it was NOT checking the King, so it must have promoted on a1, which we know isn't true.

          • King and Rook are out of the question.

          Thus, Black made the last move. Which piece moved last? Clearly it wasn't that Black rook. And if it was some other piece, it still begs the question: What was White's move before that? This is problematic because if Black's last move was, say, x.. Nb8, then we can apply the same above argument to conclude that White couldn't have made the last move.



          This can only mean that Black captured a White piece on the last move. At this point, it's pretty clear that this can only be the White light-squared Bishop. It's evident that the only square it could have been captured on is g8. But neither the Black f nor h pawns have moved, so this Bishop couldn't possibly have gotten there. Contradiction. Thus, White cannot castle.






          share|improve this answer











          $endgroup$












          • $begingroup$
            See @bass's answer.
            $endgroup$
            – Duck
            1 hour ago















          3












          $begingroup$

          (Edit: so this is wrong..)



          Brilliant puzzle! The answer is:




          No! White can't castle.




          [Spoiler alert! Scroll down at your own risk]











          We proceed by contradiction. Assume that indeed, White can castle. We have the following undeniable facts:



          • Neither the White King nor the White h-Rook have moved.

          • Since neither of them have moved, the only way the Black Rook could have gotten to a1 is via a promotion.

          • The only Black pawn that ever moved was the Black g-pawn. This is the piece that promoted.

          We now ask: On which square did this pawn promote? It's not so obvious!



          • The pawn could not have promoted on a1, because this would require 6 captures. Exactly 6 white pieces are missing. That means the pawn captured all of them. But this is impossible because the pawn can only capture on dark squares, and one of the captured pieces would have been a light-squared Bishop.

          • The pawn theoretically could have promoted on b1 via 5 captures.

          • The pawn could not have promoted on c1, for the same reason as that for a1.

          • The path could not have promoted on d1 or f1, because it has to promote to a Rook, and this would check the king, and either the rook would be taken or the King would have to move.

          • Obviously, it could not have promoted on e1.

          • It could not have promoted on g1, because then it cannot get to a1 without the King moving.

          • Obviously, it could not have promoted on h1.

          We conclude that the pawn promoted on b1 via 5 captures. Each of these captures occurred on a dark square, so the five pieces captured were:
          1. The a-Rook
          2. The dark-squared Bishop
          3. The Queen
          4. A Knight
          5. The h-pawn



          The first four captures are easy enough. The problem is the h-pawn. If this pawn stays on its column, it could never be captured by the Black g-pawn in the south-west direction. So it changed columns. To change columns, it must capture a piece. That means that the White h-pawn captured the Black Rook. Even so, this isn't enough to get captured by the Black g-pawn. We conclude that the White h-pawn captured the Black Rook and then promoted on g8 to a capturable piece, say, a second Queen.



          Ok, so the Black g-pawn captures all those 5 pieces. Here's where we're basically at:



          lol



          Notice that I'm keeping the White light-squared Bishop alive. This is very important.



          See, here's the problem now: White has only two movable pieces left: The Knight and the light-squared Bishop. Somehow, Black was able to 1) Promote on b1 to a Rook, 2) Move it to a1, and then 3) White was able to move the White Knight to b1. If the light-squared Bishop was dead, this would be impossible, because the White King would be in check while we were doing all the maneuvering! It couldn't have just been the White Knight blocking the Rook, because then it would be pinned!



          The solution? The White Bishop covers the King by moving to d1. This fact is undeniable, and we will use it very soon.



          (Edit: Could it have instead been a Black piece protecting the White King, say, the Bishop on f8? Actually yes, but this issue resolved itself, see next edit)



          Now we're all set for the main argument. The main problem that is not immediately obvious is that of the last move.



          Looking back at the original board, we first ask: Who moved last? If White moved last, what piece was moved?



          • Clearly it wasn't the b-pawn, because the Black pawn had to get in somehow.

          • It wasn't the e-pawn for sure, because the light-squared Bishop had to get out at some point in order to protect the King from the promoted Rook's check. (Edit:
            So back to the question of if it was a Black piece blocking the check. Well, then, if it was the e-pawn that moved last, then the White light-squared Bishop would be stuck. In short, there wouldn't be enough time for Black to capture this Bishop and run back to its starting square, because White can't move anything else at this point without ruining ability to castle or ruining the Black Rook-White Knight duo on a1 and b1.)

          • It couldn't have been the Knight, because then it would be moving to block the check, i.e. when the Rook got to a1 it checked the king. That means on the move before it was NOT checking the King, so it must have promoted on a1, which we know isn't true.

          • King and Rook are out of the question.

          Thus, Black made the last move. Which piece moved last? Clearly it wasn't that Black rook. And if it was some other piece, it still begs the question: What was White's move before that? This is problematic because if Black's last move was, say, x.. Nb8, then we can apply the same above argument to conclude that White couldn't have made the last move.



          This can only mean that Black captured a White piece on the last move. At this point, it's pretty clear that this can only be the White light-squared Bishop. It's evident that the only square it could have been captured on is g8. But neither the Black f nor h pawns have moved, so this Bishop couldn't possibly have gotten there. Contradiction. Thus, White cannot castle.






          share|improve this answer











          $endgroup$












          • $begingroup$
            See @bass's answer.
            $endgroup$
            – Duck
            1 hour ago













          3












          3








          3





          $begingroup$

          (Edit: so this is wrong..)



          Brilliant puzzle! The answer is:




          No! White can't castle.




          [Spoiler alert! Scroll down at your own risk]











          We proceed by contradiction. Assume that indeed, White can castle. We have the following undeniable facts:



          • Neither the White King nor the White h-Rook have moved.

          • Since neither of them have moved, the only way the Black Rook could have gotten to a1 is via a promotion.

          • The only Black pawn that ever moved was the Black g-pawn. This is the piece that promoted.

          We now ask: On which square did this pawn promote? It's not so obvious!



          • The pawn could not have promoted on a1, because this would require 6 captures. Exactly 6 white pieces are missing. That means the pawn captured all of them. But this is impossible because the pawn can only capture on dark squares, and one of the captured pieces would have been a light-squared Bishop.

          • The pawn theoretically could have promoted on b1 via 5 captures.

          • The pawn could not have promoted on c1, for the same reason as that for a1.

          • The path could not have promoted on d1 or f1, because it has to promote to a Rook, and this would check the king, and either the rook would be taken or the King would have to move.

          • Obviously, it could not have promoted on e1.

          • It could not have promoted on g1, because then it cannot get to a1 without the King moving.

          • Obviously, it could not have promoted on h1.

          We conclude that the pawn promoted on b1 via 5 captures. Each of these captures occurred on a dark square, so the five pieces captured were:
          1. The a-Rook
          2. The dark-squared Bishop
          3. The Queen
          4. A Knight
          5. The h-pawn



          The first four captures are easy enough. The problem is the h-pawn. If this pawn stays on its column, it could never be captured by the Black g-pawn in the south-west direction. So it changed columns. To change columns, it must capture a piece. That means that the White h-pawn captured the Black Rook. Even so, this isn't enough to get captured by the Black g-pawn. We conclude that the White h-pawn captured the Black Rook and then promoted on g8 to a capturable piece, say, a second Queen.



          Ok, so the Black g-pawn captures all those 5 pieces. Here's where we're basically at:



          lol



          Notice that I'm keeping the White light-squared Bishop alive. This is very important.



          See, here's the problem now: White has only two movable pieces left: The Knight and the light-squared Bishop. Somehow, Black was able to 1) Promote on b1 to a Rook, 2) Move it to a1, and then 3) White was able to move the White Knight to b1. If the light-squared Bishop was dead, this would be impossible, because the White King would be in check while we were doing all the maneuvering! It couldn't have just been the White Knight blocking the Rook, because then it would be pinned!



          The solution? The White Bishop covers the King by moving to d1. This fact is undeniable, and we will use it very soon.



          (Edit: Could it have instead been a Black piece protecting the White King, say, the Bishop on f8? Actually yes, but this issue resolved itself, see next edit)



          Now we're all set for the main argument. The main problem that is not immediately obvious is that of the last move.



          Looking back at the original board, we first ask: Who moved last? If White moved last, what piece was moved?



          • Clearly it wasn't the b-pawn, because the Black pawn had to get in somehow.

          • It wasn't the e-pawn for sure, because the light-squared Bishop had to get out at some point in order to protect the King from the promoted Rook's check. (Edit:
            So back to the question of if it was a Black piece blocking the check. Well, then, if it was the e-pawn that moved last, then the White light-squared Bishop would be stuck. In short, there wouldn't be enough time for Black to capture this Bishop and run back to its starting square, because White can't move anything else at this point without ruining ability to castle or ruining the Black Rook-White Knight duo on a1 and b1.)

          • It couldn't have been the Knight, because then it would be moving to block the check, i.e. when the Rook got to a1 it checked the king. That means on the move before it was NOT checking the King, so it must have promoted on a1, which we know isn't true.

          • King and Rook are out of the question.

          Thus, Black made the last move. Which piece moved last? Clearly it wasn't that Black rook. And if it was some other piece, it still begs the question: What was White's move before that? This is problematic because if Black's last move was, say, x.. Nb8, then we can apply the same above argument to conclude that White couldn't have made the last move.



          This can only mean that Black captured a White piece on the last move. At this point, it's pretty clear that this can only be the White light-squared Bishop. It's evident that the only square it could have been captured on is g8. But neither the Black f nor h pawns have moved, so this Bishop couldn't possibly have gotten there. Contradiction. Thus, White cannot castle.






          share|improve this answer











          $endgroup$



          (Edit: so this is wrong..)



          Brilliant puzzle! The answer is:




          No! White can't castle.




          [Spoiler alert! Scroll down at your own risk]











          We proceed by contradiction. Assume that indeed, White can castle. We have the following undeniable facts:



          • Neither the White King nor the White h-Rook have moved.

          • Since neither of them have moved, the only way the Black Rook could have gotten to a1 is via a promotion.

          • The only Black pawn that ever moved was the Black g-pawn. This is the piece that promoted.

          We now ask: On which square did this pawn promote? It's not so obvious!



          • The pawn could not have promoted on a1, because this would require 6 captures. Exactly 6 white pieces are missing. That means the pawn captured all of them. But this is impossible because the pawn can only capture on dark squares, and one of the captured pieces would have been a light-squared Bishop.

          • The pawn theoretically could have promoted on b1 via 5 captures.

          • The pawn could not have promoted on c1, for the same reason as that for a1.

          • The path could not have promoted on d1 or f1, because it has to promote to a Rook, and this would check the king, and either the rook would be taken or the King would have to move.

          • Obviously, it could not have promoted on e1.

          • It could not have promoted on g1, because then it cannot get to a1 without the King moving.

          • Obviously, it could not have promoted on h1.

          We conclude that the pawn promoted on b1 via 5 captures. Each of these captures occurred on a dark square, so the five pieces captured were:
          1. The a-Rook
          2. The dark-squared Bishop
          3. The Queen
          4. A Knight
          5. The h-pawn



          The first four captures are easy enough. The problem is the h-pawn. If this pawn stays on its column, it could never be captured by the Black g-pawn in the south-west direction. So it changed columns. To change columns, it must capture a piece. That means that the White h-pawn captured the Black Rook. Even so, this isn't enough to get captured by the Black g-pawn. We conclude that the White h-pawn captured the Black Rook and then promoted on g8 to a capturable piece, say, a second Queen.



          Ok, so the Black g-pawn captures all those 5 pieces. Here's where we're basically at:



          lol



          Notice that I'm keeping the White light-squared Bishop alive. This is very important.



          See, here's the problem now: White has only two movable pieces left: The Knight and the light-squared Bishop. Somehow, Black was able to 1) Promote on b1 to a Rook, 2) Move it to a1, and then 3) White was able to move the White Knight to b1. If the light-squared Bishop was dead, this would be impossible, because the White King would be in check while we were doing all the maneuvering! It couldn't have just been the White Knight blocking the Rook, because then it would be pinned!



          The solution? The White Bishop covers the King by moving to d1. This fact is undeniable, and we will use it very soon.



          (Edit: Could it have instead been a Black piece protecting the White King, say, the Bishop on f8? Actually yes, but this issue resolved itself, see next edit)



          Now we're all set for the main argument. The main problem that is not immediately obvious is that of the last move.



          Looking back at the original board, we first ask: Who moved last? If White moved last, what piece was moved?



          • Clearly it wasn't the b-pawn, because the Black pawn had to get in somehow.

          • It wasn't the e-pawn for sure, because the light-squared Bishop had to get out at some point in order to protect the King from the promoted Rook's check. (Edit:
            So back to the question of if it was a Black piece blocking the check. Well, then, if it was the e-pawn that moved last, then the White light-squared Bishop would be stuck. In short, there wouldn't be enough time for Black to capture this Bishop and run back to its starting square, because White can't move anything else at this point without ruining ability to castle or ruining the Black Rook-White Knight duo on a1 and b1.)

          • It couldn't have been the Knight, because then it would be moving to block the check, i.e. when the Rook got to a1 it checked the king. That means on the move before it was NOT checking the King, so it must have promoted on a1, which we know isn't true.

          • King and Rook are out of the question.

          Thus, Black made the last move. Which piece moved last? Clearly it wasn't that Black rook. And if it was some other piece, it still begs the question: What was White's move before that? This is problematic because if Black's last move was, say, x.. Nb8, then we can apply the same above argument to conclude that White couldn't have made the last move.



          This can only mean that Black captured a White piece on the last move. At this point, it's pretty clear that this can only be the White light-squared Bishop. It's evident that the only square it could have been captured on is g8. But neither the Black f nor h pawns have moved, so this Bishop couldn't possibly have gotten there. Contradiction. Thus, White cannot castle.







          share|improve this answer














          share|improve this answer



          share|improve this answer








          edited 1 hour ago

























          answered 6 hours ago









          greenturtle3141greenturtle3141

          6,33112256




          6,33112256











          • $begingroup$
            See @bass's answer.
            $endgroup$
            – Duck
            1 hour ago
















          • $begingroup$
            See @bass's answer.
            $endgroup$
            – Duck
            1 hour ago















          $begingroup$
          See @bass's answer.
          $endgroup$
          – Duck
          1 hour ago




          $begingroup$
          See @bass's answer.
          $endgroup$
          – Duck
          1 hour ago

















          draft saved

          draft discarded
















































          Thanks for contributing an answer to Puzzling Stack Exchange!


          • Please be sure to answer the question. Provide details and share your research!

          But avoid


          • Asking for help, clarification, or responding to other answers.

          • Making statements based on opinion; back them up with references or personal experience.

          Use MathJax to format equations. MathJax reference.


          To learn more, see our tips on writing great answers.




          draft saved


          draft discarded














          StackExchange.ready(
          function ()
          StackExchange.openid.initPostLogin('.new-post-login', 'https%3a%2f%2fpuzzling.stackexchange.com%2fquestions%2f85221%2fcan-white-castle%23new-answer', 'question_page');

          );

          Post as a guest















          Required, but never shown





















































          Required, but never shown














          Required, but never shown












          Required, but never shown







          Required, but never shown

































          Required, but never shown














          Required, but never shown












          Required, but never shown







          Required, but never shown







          Popular posts from this blog

          Invision Community Contents History See also References External links Navigation menuProprietaryinvisioncommunity.comIPS Community ForumsIPS Community Forumsthis blog entry"License Changes, IP.Board 3.4, and the Future""Interview -- Matt Mecham of Ibforums""CEO Invision Power Board, Matt Mecham Is a Liar, Thief!"IPB License Explanation 1.3, 1.3.1, 2.0, and 2.1ArchivedSecurity Fixes, Updates And Enhancements For IPB 1.3.1Archived"New Demo Accounts - Invision Power Services"the original"New Default Skin"the original"Invision Power Board 3.0.0 and Applications Released"the original"Archived copy"the original"Perpetual licenses being done away with""Release Notes - Invision Power Services""Introducing: IPS Community Suite 4!"Invision Community Release Notes

          Canceling a color specificationRandomly assigning color to Graphics3D objects?Default color for Filling in Mathematica 9Coloring specific elements of sets with a prime modified order in an array plotHow to pick a color differing significantly from the colors already in a given color list?Detection of the text colorColor numbers based on their valueCan color schemes for use with ColorData include opacity specification?My dynamic color schemes

          Tom Holland Mục lục Đầu đời và giáo dục | Sự nghiệp | Cuộc sống cá nhân | Phim tham gia | Giải thưởng và đề cử | Chú thích | Liên kết ngoài | Trình đơn chuyển hướngProfile“Person Details for Thomas Stanley Holland, "England and Wales Birth Registration Index, 1837-2008" — FamilySearch.org”"Meet Tom Holland... the 16-year-old star of The Impossible""Schoolboy actor Tom Holland finds himself in Oscar contention for role in tsunami drama"“Naomi Watts on the Prince William and Harry's reaction to her film about the late Princess Diana”lưu trữ"Holland and Pflueger Are West End's Two New 'Billy Elliots'""I'm so envious of my son, the movie star! British writer Dominic Holland's spent 20 years trying to crack Hollywood - but he's been beaten to it by a very unlikely rival"“Richard and Margaret Povey of Jersey, Channel Islands, UK: Information about Thomas Stanley Holland”"Tom Holland to play Billy Elliot""New Billy Elliot leaving the garage"Billy Elliot the Musical - Tom Holland - Billy"A Tale of four Billys: Tom Holland""The Feel Good Factor""Thames Christian College schoolboys join Myleene Klass for The Feelgood Factor""Government launches £600,000 arts bursaries pilot""BILLY's Chapman, Holland, Gardner & Jackson-Keen Visit Prime Minister""Elton John 'blown away' by Billy Elliot fifth birthday" (video with John's interview and fragments of Holland's performance)"First News interviews Arrietty's Tom Holland"“33rd Critics' Circle Film Awards winners”“National Board of Review Current Awards”Bản gốc"Ron Howard Whaling Tale 'In The Heart Of The Sea' Casts Tom Holland"“'Spider-Man' Finds Tom Holland to Star as New Web-Slinger”lưu trữ“Captain America: Civil War (2016)”“Film Review: ‘Captain America: Civil War’”lưu trữ“‘Captain America: Civil War’ review: Choose your own avenger”lưu trữ“The Lost City of Z reviews”“Sony Pictures and Marvel Studios Find Their 'Spider-Man' Star and Director”“‘Mary Magdalene’, ‘Current War’ & ‘Wind River’ Get 2017 Release Dates From Weinstein”“Lionsgate Unleashing Daisy Ridley & Tom Holland Starrer ‘Chaos Walking’ In Cannes”“PTA's 'Master' Leads Chicago Film Critics Nominations, UPDATED: Houston and Indiana Critics Nominations”“Nominaciones Goya 2013 Telecinco Cinema – ENG”“Jameson Empire Film Awards: Martin Freeman wins best actor for performance in The Hobbit”“34th Annual Young Artist Awards”Bản gốc“Teen Choice Awards 2016—Captain America: Civil War Leads Second Wave of Nominations”“BAFTA Film Award Nominations: ‘La La Land’ Leads Race”“Saturn Awards Nominations 2017: 'Rogue One,' 'Walking Dead' Lead”Tom HollandTom HollandTom HollandTom Hollandmedia.gettyimages.comWorldCat Identities300279794no20130442900000 0004 0355 42791085670554170004732cb16706349t(data)XX5557367